Are the partial sums for $sum_{n=1}^{infty}sin(n^a)$ bounded for $ageq1$ and unbounded for $0<a<1$?











up vote
2
down vote

favorite












I know that the partial sums of $$sum_{n=1}^{infty}sin(n)$$
are bounded between $frac{cosleft(frac{1}{2}right)-1}{2sinleft(frac{1}{2} right)}$ and $frac{1+cosleft(frac{1}{2} right)}{2sinleft(frac{1}{2}right)}$.



On the other hand, the partial sums of
$$sum_{n=1}^{infty}sin(sqrt{n})$$
are unbounded.



I think that the partial sums of $sum_{n=1}^{infty}sin(n^a)$ are bounded for $a geq 1$ and unbounded for $0< a<1$, but how can I prove this? I think this question involves Euler-Maclaurin sum.










share|cite|improve this question
























  • You mean the partial sums of these series are bounded/unbounded.
    – zhw.
    Nov 10 at 20:12






  • 1




    It would be good to edit to reflect this.
    – zhw.
    Nov 10 at 20:26






  • 1




    The partial sums of $sum sin(n^alpha)$ are not bounded for $alpha>1$, but $sum_{n=1}^{N}sin(n^alpha)$ can be suitably controlled by a power of $N$ times a power of $log N$. See Weyl's inequality, giving $$sum_{n=1}^{N}sin(n^2) ll sqrt{N}log^2 N,$$ for instance.
    – Jack D'Aurizio
    Nov 10 at 22:11








  • 1




    If you could achieve such bound $sqrt{N}log^2 N$, it solves your first question in math.stackexchange.com/questions/215528/…. When I tried it years ago, it did not work out very well, I could obtain $alphaleq 7/8$. If you managed to prove such bound, please include your method as an answer.
    – i707107
    Nov 11 at 3:51






  • 1




    @JackD'Aurizio You mentioned that the partial sums are unbounded when $a>1$. Would you please include that as an answer? I know Terry Tao solved for $a>1$ integer case in here mathoverflow.net/questions/201250/…
    – i707107
    Nov 12 at 18:14















up vote
2
down vote

favorite












I know that the partial sums of $$sum_{n=1}^{infty}sin(n)$$
are bounded between $frac{cosleft(frac{1}{2}right)-1}{2sinleft(frac{1}{2} right)}$ and $frac{1+cosleft(frac{1}{2} right)}{2sinleft(frac{1}{2}right)}$.



On the other hand, the partial sums of
$$sum_{n=1}^{infty}sin(sqrt{n})$$
are unbounded.



I think that the partial sums of $sum_{n=1}^{infty}sin(n^a)$ are bounded for $a geq 1$ and unbounded for $0< a<1$, but how can I prove this? I think this question involves Euler-Maclaurin sum.










share|cite|improve this question
























  • You mean the partial sums of these series are bounded/unbounded.
    – zhw.
    Nov 10 at 20:12






  • 1




    It would be good to edit to reflect this.
    – zhw.
    Nov 10 at 20:26






  • 1




    The partial sums of $sum sin(n^alpha)$ are not bounded for $alpha>1$, but $sum_{n=1}^{N}sin(n^alpha)$ can be suitably controlled by a power of $N$ times a power of $log N$. See Weyl's inequality, giving $$sum_{n=1}^{N}sin(n^2) ll sqrt{N}log^2 N,$$ for instance.
    – Jack D'Aurizio
    Nov 10 at 22:11








  • 1




    If you could achieve such bound $sqrt{N}log^2 N$, it solves your first question in math.stackexchange.com/questions/215528/…. When I tried it years ago, it did not work out very well, I could obtain $alphaleq 7/8$. If you managed to prove such bound, please include your method as an answer.
    – i707107
    Nov 11 at 3:51






  • 1




    @JackD'Aurizio You mentioned that the partial sums are unbounded when $a>1$. Would you please include that as an answer? I know Terry Tao solved for $a>1$ integer case in here mathoverflow.net/questions/201250/…
    – i707107
    Nov 12 at 18:14













up vote
2
down vote

favorite









up vote
2
down vote

favorite











I know that the partial sums of $$sum_{n=1}^{infty}sin(n)$$
are bounded between $frac{cosleft(frac{1}{2}right)-1}{2sinleft(frac{1}{2} right)}$ and $frac{1+cosleft(frac{1}{2} right)}{2sinleft(frac{1}{2}right)}$.



On the other hand, the partial sums of
$$sum_{n=1}^{infty}sin(sqrt{n})$$
are unbounded.



I think that the partial sums of $sum_{n=1}^{infty}sin(n^a)$ are bounded for $a geq 1$ and unbounded for $0< a<1$, but how can I prove this? I think this question involves Euler-Maclaurin sum.










share|cite|improve this question















I know that the partial sums of $$sum_{n=1}^{infty}sin(n)$$
are bounded between $frac{cosleft(frac{1}{2}right)-1}{2sinleft(frac{1}{2} right)}$ and $frac{1+cosleft(frac{1}{2} right)}{2sinleft(frac{1}{2}right)}$.



On the other hand, the partial sums of
$$sum_{n=1}^{infty}sin(sqrt{n})$$
are unbounded.



I think that the partial sums of $sum_{n=1}^{infty}sin(n^a)$ are bounded for $a geq 1$ and unbounded for $0< a<1$, but how can I prove this? I think this question involves Euler-Maclaurin sum.







calculus sequences-and-series trigonometric-series euler-maclaurin






share|cite|improve this question















share|cite|improve this question













share|cite|improve this question




share|cite|improve this question








edited Nov 10 at 22:22









zhw.

70.4k42975




70.4k42975










asked Nov 10 at 20:05









Larry

1,1021622




1,1021622












  • You mean the partial sums of these series are bounded/unbounded.
    – zhw.
    Nov 10 at 20:12






  • 1




    It would be good to edit to reflect this.
    – zhw.
    Nov 10 at 20:26






  • 1




    The partial sums of $sum sin(n^alpha)$ are not bounded for $alpha>1$, but $sum_{n=1}^{N}sin(n^alpha)$ can be suitably controlled by a power of $N$ times a power of $log N$. See Weyl's inequality, giving $$sum_{n=1}^{N}sin(n^2) ll sqrt{N}log^2 N,$$ for instance.
    – Jack D'Aurizio
    Nov 10 at 22:11








  • 1




    If you could achieve such bound $sqrt{N}log^2 N$, it solves your first question in math.stackexchange.com/questions/215528/…. When I tried it years ago, it did not work out very well, I could obtain $alphaleq 7/8$. If you managed to prove such bound, please include your method as an answer.
    – i707107
    Nov 11 at 3:51






  • 1




    @JackD'Aurizio You mentioned that the partial sums are unbounded when $a>1$. Would you please include that as an answer? I know Terry Tao solved for $a>1$ integer case in here mathoverflow.net/questions/201250/…
    – i707107
    Nov 12 at 18:14


















  • You mean the partial sums of these series are bounded/unbounded.
    – zhw.
    Nov 10 at 20:12






  • 1




    It would be good to edit to reflect this.
    – zhw.
    Nov 10 at 20:26






  • 1




    The partial sums of $sum sin(n^alpha)$ are not bounded for $alpha>1$, but $sum_{n=1}^{N}sin(n^alpha)$ can be suitably controlled by a power of $N$ times a power of $log N$. See Weyl's inequality, giving $$sum_{n=1}^{N}sin(n^2) ll sqrt{N}log^2 N,$$ for instance.
    – Jack D'Aurizio
    Nov 10 at 22:11








  • 1




    If you could achieve such bound $sqrt{N}log^2 N$, it solves your first question in math.stackexchange.com/questions/215528/…. When I tried it years ago, it did not work out very well, I could obtain $alphaleq 7/8$. If you managed to prove such bound, please include your method as an answer.
    – i707107
    Nov 11 at 3:51






  • 1




    @JackD'Aurizio You mentioned that the partial sums are unbounded when $a>1$. Would you please include that as an answer? I know Terry Tao solved for $a>1$ integer case in here mathoverflow.net/questions/201250/…
    – i707107
    Nov 12 at 18:14
















You mean the partial sums of these series are bounded/unbounded.
– zhw.
Nov 10 at 20:12




You mean the partial sums of these series are bounded/unbounded.
– zhw.
Nov 10 at 20:12




1




1




It would be good to edit to reflect this.
– zhw.
Nov 10 at 20:26




It would be good to edit to reflect this.
– zhw.
Nov 10 at 20:26




1




1




The partial sums of $sum sin(n^alpha)$ are not bounded for $alpha>1$, but $sum_{n=1}^{N}sin(n^alpha)$ can be suitably controlled by a power of $N$ times a power of $log N$. See Weyl's inequality, giving $$sum_{n=1}^{N}sin(n^2) ll sqrt{N}log^2 N,$$ for instance.
– Jack D'Aurizio
Nov 10 at 22:11






The partial sums of $sum sin(n^alpha)$ are not bounded for $alpha>1$, but $sum_{n=1}^{N}sin(n^alpha)$ can be suitably controlled by a power of $N$ times a power of $log N$. See Weyl's inequality, giving $$sum_{n=1}^{N}sin(n^2) ll sqrt{N}log^2 N,$$ for instance.
– Jack D'Aurizio
Nov 10 at 22:11






1




1




If you could achieve such bound $sqrt{N}log^2 N$, it solves your first question in math.stackexchange.com/questions/215528/…. When I tried it years ago, it did not work out very well, I could obtain $alphaleq 7/8$. If you managed to prove such bound, please include your method as an answer.
– i707107
Nov 11 at 3:51




If you could achieve such bound $sqrt{N}log^2 N$, it solves your first question in math.stackexchange.com/questions/215528/…. When I tried it years ago, it did not work out very well, I could obtain $alphaleq 7/8$. If you managed to prove such bound, please include your method as an answer.
– i707107
Nov 11 at 3:51




1




1




@JackD'Aurizio You mentioned that the partial sums are unbounded when $a>1$. Would you please include that as an answer? I know Terry Tao solved for $a>1$ integer case in here mathoverflow.net/questions/201250/…
– i707107
Nov 12 at 18:14




@JackD'Aurizio You mentioned that the partial sums are unbounded when $a>1$. Would you please include that as an answer? I know Terry Tao solved for $a>1$ integer case in here mathoverflow.net/questions/201250/…
– i707107
Nov 12 at 18:14










1 Answer
1






active

oldest

votes

















up vote
1
down vote



accepted










If $a>1$ is an integer, then this post in MO by Terry Tao solves that the partial sums are not bounded.



If $a>1$ is not an integer, the same argument by Terry Tao still works, since we have equidistribution of $n^a$ mod $2pi$, and any sum or difference of $(n+i)^a$ with $1leq i leq h$ modulo $2pi$. Let $X_i$ be the random variable $sin^a (k+i)$ where $k=1,ldots n$. Assuming the boundedness of partial sums, we end up having a contradiction that the random variables $X_1, ldots, X_h$ such that $mathrm{Var}(X_1+cdots+X_h)$ is bounded as $hrightarrow infty$ by assumption, but $mathrm{Var}(X_1+cdots+X_h)sim h/2$ as $hrightarrowinfty$.






share|cite|improve this answer























    Your Answer





    StackExchange.ifUsing("editor", function () {
    return StackExchange.using("mathjaxEditing", function () {
    StackExchange.MarkdownEditor.creationCallbacks.add(function (editor, postfix) {
    StackExchange.mathjaxEditing.prepareWmdForMathJax(editor, postfix, [["$", "$"], ["\\(","\\)"]]);
    });
    });
    }, "mathjax-editing");

    StackExchange.ready(function() {
    var channelOptions = {
    tags: "".split(" "),
    id: "69"
    };
    initTagRenderer("".split(" "), "".split(" "), channelOptions);

    StackExchange.using("externalEditor", function() {
    // Have to fire editor after snippets, if snippets enabled
    if (StackExchange.settings.snippets.snippetsEnabled) {
    StackExchange.using("snippets", function() {
    createEditor();
    });
    }
    else {
    createEditor();
    }
    });

    function createEditor() {
    StackExchange.prepareEditor({
    heartbeatType: 'answer',
    convertImagesToLinks: true,
    noModals: true,
    showLowRepImageUploadWarning: true,
    reputationToPostImages: 10,
    bindNavPrevention: true,
    postfix: "",
    imageUploader: {
    brandingHtml: "Powered by u003ca class="icon-imgur-white" href="https://imgur.com/"u003eu003c/au003e",
    contentPolicyHtml: "User contributions licensed under u003ca href="https://creativecommons.org/licenses/by-sa/3.0/"u003ecc by-sa 3.0 with attribution requiredu003c/au003e u003ca href="https://stackoverflow.com/legal/content-policy"u003e(content policy)u003c/au003e",
    allowUrls: true
    },
    noCode: true, onDemand: true,
    discardSelector: ".discard-answer"
    ,immediatelyShowMarkdownHelp:true
    });


    }
    });














     

    draft saved


    draft discarded


















    StackExchange.ready(
    function () {
    StackExchange.openid.initPostLogin('.new-post-login', 'https%3a%2f%2fmath.stackexchange.com%2fquestions%2f2993073%2fare-the-partial-sums-for-sum-n-1-infty-sinna-bounded-for-a-geq1-and%23new-answer', 'question_page');
    }
    );

    Post as a guest















    Required, but never shown

























    1 Answer
    1






    active

    oldest

    votes








    1 Answer
    1






    active

    oldest

    votes









    active

    oldest

    votes






    active

    oldest

    votes








    up vote
    1
    down vote



    accepted










    If $a>1$ is an integer, then this post in MO by Terry Tao solves that the partial sums are not bounded.



    If $a>1$ is not an integer, the same argument by Terry Tao still works, since we have equidistribution of $n^a$ mod $2pi$, and any sum or difference of $(n+i)^a$ with $1leq i leq h$ modulo $2pi$. Let $X_i$ be the random variable $sin^a (k+i)$ where $k=1,ldots n$. Assuming the boundedness of partial sums, we end up having a contradiction that the random variables $X_1, ldots, X_h$ such that $mathrm{Var}(X_1+cdots+X_h)$ is bounded as $hrightarrow infty$ by assumption, but $mathrm{Var}(X_1+cdots+X_h)sim h/2$ as $hrightarrowinfty$.






    share|cite|improve this answer



























      up vote
      1
      down vote



      accepted










      If $a>1$ is an integer, then this post in MO by Terry Tao solves that the partial sums are not bounded.



      If $a>1$ is not an integer, the same argument by Terry Tao still works, since we have equidistribution of $n^a$ mod $2pi$, and any sum or difference of $(n+i)^a$ with $1leq i leq h$ modulo $2pi$. Let $X_i$ be the random variable $sin^a (k+i)$ where $k=1,ldots n$. Assuming the boundedness of partial sums, we end up having a contradiction that the random variables $X_1, ldots, X_h$ such that $mathrm{Var}(X_1+cdots+X_h)$ is bounded as $hrightarrow infty$ by assumption, but $mathrm{Var}(X_1+cdots+X_h)sim h/2$ as $hrightarrowinfty$.






      share|cite|improve this answer

























        up vote
        1
        down vote



        accepted







        up vote
        1
        down vote



        accepted






        If $a>1$ is an integer, then this post in MO by Terry Tao solves that the partial sums are not bounded.



        If $a>1$ is not an integer, the same argument by Terry Tao still works, since we have equidistribution of $n^a$ mod $2pi$, and any sum or difference of $(n+i)^a$ with $1leq i leq h$ modulo $2pi$. Let $X_i$ be the random variable $sin^a (k+i)$ where $k=1,ldots n$. Assuming the boundedness of partial sums, we end up having a contradiction that the random variables $X_1, ldots, X_h$ such that $mathrm{Var}(X_1+cdots+X_h)$ is bounded as $hrightarrow infty$ by assumption, but $mathrm{Var}(X_1+cdots+X_h)sim h/2$ as $hrightarrowinfty$.






        share|cite|improve this answer














        If $a>1$ is an integer, then this post in MO by Terry Tao solves that the partial sums are not bounded.



        If $a>1$ is not an integer, the same argument by Terry Tao still works, since we have equidistribution of $n^a$ mod $2pi$, and any sum or difference of $(n+i)^a$ with $1leq i leq h$ modulo $2pi$. Let $X_i$ be the random variable $sin^a (k+i)$ where $k=1,ldots n$. Assuming the boundedness of partial sums, we end up having a contradiction that the random variables $X_1, ldots, X_h$ such that $mathrm{Var}(X_1+cdots+X_h)$ is bounded as $hrightarrow infty$ by assumption, but $mathrm{Var}(X_1+cdots+X_h)sim h/2$ as $hrightarrowinfty$.







        share|cite|improve this answer














        share|cite|improve this answer



        share|cite|improve this answer








        edited Nov 14 at 5:17

























        answered Nov 14 at 5:09









        i707107

        11.8k21447




        11.8k21447






























             

            draft saved


            draft discarded



















































             


            draft saved


            draft discarded














            StackExchange.ready(
            function () {
            StackExchange.openid.initPostLogin('.new-post-login', 'https%3a%2f%2fmath.stackexchange.com%2fquestions%2f2993073%2fare-the-partial-sums-for-sum-n-1-infty-sinna-bounded-for-a-geq1-and%23new-answer', 'question_page');
            }
            );

            Post as a guest















            Required, but never shown





















































            Required, but never shown














            Required, but never shown












            Required, but never shown







            Required, but never shown

































            Required, but never shown














            Required, but never shown












            Required, but never shown







            Required, but never shown







            Popular posts from this blog

            Biblatex bibliography style without URLs when DOI exists (in Overleaf with Zotero bibliography)

            ComboBox Display Member on multiple fields

            Is it possible to collect Nectar points via Trainline?